Stay ahead of learning milestones! Enroll in a class over the summer!

G
Topic
First Poster
Last Poster
No topics here!
Putnam 2015 A3
Kent Merryfield   29
N Feb 20, 2025 by smileapple
Compute \[\log_2\left(\prod_{a=1}^{2015}\prod_{b=1}^{2015}\left(1+e^{2\pi iab/2015}\right)\right)\]Here $i$ is the imaginary unit (that is, $i^2=-1$).
29 replies
Kent Merryfield
Dec 6, 2015
smileapple
Feb 20, 2025
Putnam 2015 A3
G H J
The post below has been deleted. Click to close.
This post has been deleted. Click here to see post.
Kent Merryfield
18574 posts
#1 • 4 Y
Y by opptoinfinity, Davi-8191, Adventure10, Mango247
Compute \[\log_2\left(\prod_{a=1}^{2015}\prod_{b=1}^{2015}\left(1+e^{2\pi iab/2015}\right)\right)\]Here $i$ is the imaginary unit (that is, $i^2=-1$).
Z K Y
The post below has been deleted. Click to close.
This post has been deleted. Click here to see post.
Kent Merryfield
18574 posts
#2 • 4 Y
Y by v_Enhance, Adventure10, Mango247, Dhruv777
The value of that logarithm is $13725.$

Let $n>1$ be an odd integer, and let $\zeta = e^{2\pi i/n}.$ Then the numbers $1+\zeta^k$ for $1\le k\le n$ are the $n$ roots of the polynomial $(z-1)^n=1.$ Multiplied out, that is $z^n - \cdots -2=0.$ The product of the roots is $2.$

If $a$ is relatively prime to $n$, then the numbers $\zeta^{ab}$ for $1\le b\le n$ encompass all $n$ of possible powers of $\zeta,$ and hence $\prod_{b=1}^n(1+\zeta^{ab})=2.$

But if $\gcd(a,n)=k>1,$ that means that $a=jk,$ with $k\mid n$ and $j$ relatively prime to $n/k.$ In that case, the powers $\zeta^{ab}$ can be written as $(\zeta^{k})^{jb}.$ We have that the numbers $(1+(\zeta^k)^{jb})$ are the roots of $(z-1)^{n/k}=1,$ listed $k$ times each. That means that $\prod_{b=1}^n(1+\zeta^{ab})=2^k.$

To assemble our answer, we note that $2015=5\cdot 13\cdot 31.$ In each line below, the ``inside product" is $\prod_{b=1}^n(1+\zeta^{ab}).$

$a$ relatively prime to $2015.$ Inside product is $2,$ and this happens $\phi(2015)=1440$ times.

$a=5j,$ $j$ relatively prime to $403.$ Inside product is $2^5,$ happens $\phi(403)=360$ times.

$a=13j,$ $j$ relatively prime to $155.$ Inside product is $2^{13},$ happens $\phi(155)=120$ times.

$a=31j,$ $j$ relatively prime to $65.$ Inside product is $2^{31},$ happens $\phi(65)=48$ times.

$a=65j,$ $j$ relatively prime to $31.$ Inside product is $2^{65},$ happens $\phi(31)=30$ times.

$a=155j,$ $j$ relatively prime to $13.$ Inside product is $2^{155},$ happens $\phi(13)=12$ times.

$a=403j,$ $j$ relatively prime to $5.$ Inside product is $2^{403},$ happens $\phi(5)=4$ times.

$a=2015.$ Inside product is $2^{2015}.$ Happens once.

Multiply all of these together and take the base 2 logarithm (meaning we add the exponents). The result is \[1440\cdot 1+360\cdot 5+120\cdot 13+48\cdot 31+30\cdot 65+12\cdot 155+4\cdot 403+1\cdot 2015 = 13725.\]
This post has been edited 1 time. Last edited by Kent Merryfield, Dec 8, 2015, 2:48 PM
Z K Y
The post below has been deleted. Click to close.
This post has been deleted. Click here to see post.
v_Enhance
6871 posts
#3 • 12 Y
Y by Kent Merryfield, GreenKeeper, rkm0959, CeuAzul, rafayaashary1, Laindingoldlee, Aryan-23, XbenX, Gaussian_cyber, inoxb198, Adventure10, Mango247
You can evaluate the last part with a trick. For completeness, full solution below:

First, we use the fact that for any odd integer $m$, we have
\[ \prod_{1 \le b \le m} (1 + \zeta^b) = 2 \]where $\zeta$ is an $m$th root of unity. (Just plug $-1$ into $X^m-1$.) Thus
\begin{align*}
	\log_2 \prod_{a=1}^{2015} \prod_{b=1}^{2015}
	\left( 1 + e^{\frac{2\pi i a b}{2015}} \right)
	&= \log_2 \prod_{a=1}^{2015} 2^{\gcd(a,2015)} \\
	&= \sum_{a=1}^{2015} \gcd(a,2015) \\
	&= \sum_{d \mid 2015} \frac{2015}{d} \phi(d) \\
	&= (5+\phi(5))(13+\phi(13))(31+\phi(31)) \\
	&= 9 \cdot 25 \cdot 61 \\
	&= 13725.
\end{align*}
Z K Y
The post below has been deleted. Click to close.
This post has been deleted. Click here to see post.
BOGTRO
5818 posts
#4 • 2 Y
Y by Adventure10, Mango247
Sol
Z K Y
The post below has been deleted. Click to close.
This post has been deleted. Click here to see post.
hoeij
35 posts
#5 • 3 Y
Y by Adventure10, Mango247, KnowingAnt
Notice that the function $f(n) := \sum_{a=1}^n {\rm gcd}(a,n)$ is also a multiplicative function. So $f(2015) = f(5) f(13) f(31)$.
This post has been edited 1 time. Last edited by hoeij, Dec 7, 2015, 2:57 PM
Reason: Replaced plain text by LaTeX
Z K Y
The post below has been deleted. Click to close.
This post has been deleted. Click here to see post.
pi37
2079 posts
#6 • 3 Y
Y by rafayaashary1, Adventure10, Mango247
Sketch:
Let
\[
F(x)=\prod_{1\le a,b\le 2015} (x-\omega^{ab})
\]where $\omega$ is a primitive $2015$th root of unity. We want to find $\log_2\left(-F(-1)\right)$. Then we can write (after checking some symmetry thing)
\[
F(x)=\prod_{d\mid 2015} \Phi_d(x)^{h(d)}
\]for some $h$ mapping from the divisors of $2015$ to $\mathbb{Z}_{\ge 0}$. But $\Phi_{d}(-1)=1$ unless $d=1$ for $d\mid 2015$, so it suffices to find $h(1)$. This is the number of pairs $(a,b)$ with $2015\mid ab$, which is easily computed to be $(2\cdot 5-1)(2\cdot 13-1)(2\cdot 31-1)=13725$. Thus our answer is
\[
\log_2\left(-(-2)^{13725}\right)=13725
\]Also, it took a lot longer than it should have to realize $403$ is not prime...
This post has been edited 1 time. Last edited by pi37, Dec 6, 2015, 11:02 PM
Z K Y
The post below has been deleted. Click to close.
This post has been deleted. Click here to see post.
Naysh
2134 posts
#7 • 3 Y
Y by KarlMahlburg, Adventure10, Mango247
Solution
Z K Y
The post below has been deleted. Click to close.
This post has been deleted. Click here to see post.
jh235
910 posts
#8 • 2 Y
Y by Adventure10, Mango247
I lot of people at the location I was at just did $$e^{\frac{2\pi i}{2015}ab}=\left(e^{2\pi i}\right)^{\frac{ab}{2015}}=1^{\frac{ab}{2015}}=1\Rightarrow$$the answer is $2015^2$. I am guessing that this ended up being a common mistake for those unfamiliar with complex numbers.
This post has been edited 2 times. Last edited by jh235, Dec 7, 2015, 12:35 AM
Z K Y
The post below has been deleted. Click to close.
This post has been deleted. Click here to see post.
AnderExtrema
417 posts
#9 • 2 Y
Y by Adventure10, Mango247
jh235 wrote:
I lot of people at the location I was at just did $$e^{\frac{2\pi i}{2015}ab}=\left(e^{2\pi i}\right)^{\frac{ab}{2015}}=1^{\frac{ab}{2015}}=1\Rightarrow$$the answer is $2015^2$.

What is actually wrong with the logic you just stated? I didn't think to do that on the exam, but looking at it I can't see why it's wrong.
Z K Y
The post below has been deleted. Click to close.
This post has been deleted. Click here to see post.
djmathman
7936 posts
#10 • 3 Y
Y by AnderExtrema, Adventure10, Mango247
First off, you're looking at $1+\omega^{ab}$, not $\omega^{ab}$, and distributive laws don't really hold for exponents :P

On a similar vein, one may ask why the product of each of the $2015$ terms formed when $a$ is fixed is not simply $2$. The basic problem with this is that the idea of the product being equal to $2$ relies on the fact that the each of the roots of unity is hit exactly once. That doesn't occur whenever $\gcd(n,2015)>1$. For example, if $n=5$, the product will cover $\omega^5$, $\omega^{10}$, $\ldots$, $\omega^{2015}$, then the sequence will repeat four more times. Each of these can be considered a primitive $403^{\text{rd}}$ root of unity, so together their product (of just these 403 terms) is 2. This means that for $n=5$ the result is $2^5$.
This post has been edited 2 times. Last edited by djmathman, Dec 7, 2015, 1:13 AM
Z K Y
The post below has been deleted. Click to close.
This post has been deleted. Click here to see post.
Kent Merryfield
18574 posts
#11 • 3 Y
Y by v_Enhance, Adventure10, Mango247
v_Enhance wrote:
You can evaluate the last part with a trick.
Thanks, v_Enhance. I knew all along that this was a convolution of multiplicative functions. I didn't have the patience to figure out which functions were involved and what their convolution was. And I paid a price for it: the number that I wrote down while proctoring was incorrect due to an arithmetic error. I only fixed it later that night when I started writing up solutions in \LaTeX in anticipation of posting them.
Z K Y
The post below has been deleted. Click to close.
This post has been deleted. Click here to see post.
Kent Merryfield
18574 posts
#12 • 4 Y
Y by GreenKeeper, Naysh, v_Enhance, Adventure10
Just so I have this written down somewhere, I want to make all of this more explicit. This has all been said by various posters above; I'm just collecting it in one place.

In number theory, a multiplicative function is a function $g$ whose domain is $\mathbb{N}$ such that $g(ab)=g(a)g(b)$ whenever $a$ and $b$ are relatively prime. A multiplicative function is determined by its values on prime powers.

Given two multiplicative functions, we may compute their convolution: \[f*g(n)=\sum_{d\mid n}f(d)g\left(\frac nd\right).\]We can show that $f*g$ is multiplicative whenever $f$ and $g$ are multiplicative.

The identity function $\mathrm{id}(n)=n$ and the Euler totient function $\phi(n)$ are both multiplicative functions. They are determined on prime powers by $\mathrm{id}(p^k)=p^k$ and $\phi(p^k)=p^{k-1}(p-1).$

Let $f(n)=\log_2\left(\prod_{a=1}^n\prod_{b=1}^n(1+e^{2\pi i ab/n})\right).$

By several different posters' arguments above (including mine), $f=\mathrm{id}*\phi.$ Since $f$ is given by a convolution of multiplicative functions, $f$ is multiplicative. Therefore it is determined by its values at prime powers. \begin{align*}f(p^k)&=\sum_{j=0}^{k}p^j\phi(p^{k-j})\\
&=\sum_{j=0}^{k-1}p^jp^{k-j-1}(p-1)+p^k\\
&=kp^{k-1}(p-1)+p^k=p^{k-1}(kp-k+p)\end{align*}Particularized to $k=1,$ we have $f(p)=2p-1,$ and that's all we needed for this particular problem.
\[f(2015)=f(5)f(13)f(31)=9\cdot 25\cdot 61=13725.\]There is a transform appropriate to this: Dirichlet series.
If $f$ is multiplicative define $F(s)=\sum_{n=1}^{\infty}\frac{f(n)}{n^s},$ for sufficiently large $s.$

If $h=f*g,$ then $H(s)=F(s)G(s).$

The Dirichlet series associated to $\mathrm{id}$ is $\zeta(s-1)$ and the Dirichlet series associated to $\phi$ is $\frac{\zeta(s-1)}{\zeta(s)}.$ Hence for the function $f$ in this problem, $F(s)=\frac{\left(\zeta(s-1)\right)^2}{\zeta(s)}.$
This post has been edited 1 time. Last edited by Kent Merryfield, Dec 7, 2015, 7:49 PM
Reason: Spelling
Z K Y
The post below has been deleted. Click to close.
This post has been deleted. Click here to see post.
themorninglighttt
789 posts
#13 • 2 Y
Y by Adventure10, Mango247
How many points off would it be if I did my calculation wrong, but I had everything else right?
Z K Y
The post below has been deleted. Click to close.
This post has been deleted. Click here to see post.
Kent Merryfield
18574 posts
#14 • 2 Y
Y by Adventure10, Mango247
Correction:

The "multiplicative function" $f(n)$ is $\mathrm{id}*\phi(n)$ for $n$ odd, but it is undefined for $n$ even. We could make a multiplicative function by defining it to be zero for $n$ even, but if we do that, those Dirichlet series I claimed above will need to be recalculated.
Z K Y
The post below has been deleted. Click to close.
This post has been deleted. Click here to see post.
mathocean97
606 posts
#15 • 2 Y
Y by Adventure10, Mango247
Are you sure? I'm pretty sure it is multiplicative everywhere.
Z K Y
The post below has been deleted. Click to close.
This post has been deleted. Click here to see post.
adityaguharoy
4655 posts
#16 • 2 Y
Y by Adventure10, Mango247
yes it is multiplicative
Z K Y
The post below has been deleted. Click to close.
This post has been deleted. Click here to see post.
adityaguharoy
4655 posts
#17 • 2 Y
Y by Adventure10, Mango247
yes multiplicative...
Z K Y
The post below has been deleted. Click to close.
This post has been deleted. Click here to see post.
AnderExtrema
417 posts
#18 • 1 Y
Y by Adventure10
I thought you learned what the edit function was. (My tone is not even sarcastic, more so just questioning.)
Z K Y
The post below has been deleted. Click to close.
This post has been deleted. Click here to see post.
Kent Merryfield
18574 posts
#19 • 2 Y
Y by Adventure10, Mango247
No, the problem with $f(n)$ as defined in my post #12 above is that it is not defined for $n$ even. A multiplicative function is supposed to be defined on $\mathbb{N}.$

If $n$ is even, then $\prod_{j=1}^n(1+e^{2\pi ij/n})=0.$ With that factor in there, the whole double product is zero, which makes its logarithm undefined.

As I said, we could make a multiplicative function by defining $f(2)=0$ and therefore $f(n)=0$ for all even $n.$
Z K Y
The post below has been deleted. Click to close.
This post has been deleted. Click here to see post.
adityaguharoy
4655 posts
#20 • 2 Y
Y by Adventure10, Mango247
yes precisely that
Z K Y
The post below has been deleted. Click to close.
This post has been deleted. Click here to see post.
UrInvalid
627 posts
#21 • 2 Y
Y by Adventure10, Mango247
So I just mocked this, and I have a question:
How many points would one get for explaining the roots thing, finding the correct final sum, and then adding wrong? (like, my solution is almost identical to post #2, but i messed up in the tens place)
Z K Y
The post below has been deleted. Click to close.
This post has been deleted. Click here to see post.
jmerry
12096 posts
#22 • 1 Y
Y by Adventure10
The standard response: Putnam grading is opaque. No commentary on it is ever released, and the individual scores you get back through your sponsor at the school aren't broken down by problem.

That said, a simple arithmetic error like that should be a $10^-$, not a $0^+$. (All scores on problems are in the 0 to 2 range or the 8 to 10 range)
Z K Y
The post below has been deleted. Click to close.
This post has been deleted. Click here to see post.
UrInvalid
627 posts
#23 • 2 Y
Y by Adventure10, Mango247
jmerry wrote:
The standard response: Putnam grading is opaque. No commentary on it is ever released, and the individual scores you get back through your sponsor at the school aren't broken down by problem.

That said, a simple arithmetic error like that should be a $10^-$, not a $0^+$. (All scores on problems are in the 0 to 2 range or the 8 to 10 range)

Cool, thank you.
Z K Y
The post below has been deleted. Click to close.
This post has been deleted. Click here to see post.
sqing
41438 posts
#24 • 1 Y
Y by Adventure10
https://artofproblemsolving.com/community/c6h1883358p12823215:
Compute \[\log_2\left(\prod_{a=0}^{2018}\prod_{b=0}^{2018}\left(1+e^{\frac{2\pi iab}{2019}}\right)\right)\]Here $i$ is the imaginary unit (that is, $i^2=-1$).
Z K Y
The post below has been deleted. Click to close.
This post has been deleted. Click here to see post.
Chimphechunu
233 posts
#25 • 2 Y
Y by Adventure10, Mango247
What is the j here?
Z K Y
The post below has been deleted. Click to close.
This post has been deleted. Click here to see post.
sixoneeight
1137 posts
#26
Y by
For a fixed $a$, let $d = \gcd(a, 2015)$. Then, the inner product contains the $\frac{2015}{d}$-th roots of unity, each occurring $d$ times. Specifically, let
\[
P_n(x) = x^n-1=\prod_{c=0}^{n-1}\left(x-e^{2\pi ic/n}\right)
\]be a monic polynomial with $n$ roots being the $n$-th roots of unity. Then,
\[
\prod_{b=1}^{2015}\left(1+e^{2\pi i ab/2015}\right) = (-1)^{2015}\left(P_{\frac{2015}{d}}(-1)\right)^{d}=2^{d}
\]Thus, the entire expression is simply
\[
\sum_{a=1}^{\infty}\gcd(a,2015) = (1+1+1+1+5)(1+1+\dots+1+13)(1+1+\dots+1+31)
\]because we have a factor of $5$ every $5$ terms and a factor of $1$ otherwise, etc. We can evaluate the answer as $\boxed{13725}$.
Z K Y
The post below has been deleted. Click to close.
This post has been deleted. Click here to see post.
lifeismathematics
1188 posts
#27 • 1 Y
Y by Sagnik123Biswas
cosnider $\zeta:=e^{\frac{2i\pi}{n}}$ then $\zeta^{k}=\zeta_{k}$ are roots of the equation $x^{n}-1$ $\forall$ $0 \leqslant k \leqslant n-1$

We notice , $$\prod_{i=1}^{n}(1+\zeta_{i})=2$$for odd $n$.

Noticing the fact that if , $\gcd(a,n)=1$ then the set $\mathcal{S}=\{0,a,2a,\cdots, (n-1)a\} \equiv \{0,1,\cdots , n-1\} \pmod{n}$

So , $$\prod_{b=1}^{n} \left(1+e^{\frac{i2\pi ab}{n}}\right)=2$$whenever $\gcd(a,n)=1$.

Now consider $\gcd(a,n):=d$ , then $$\prod_{b=1}^{\frac{2015}{d}} \displaystyle \left(1+e^{\frac{i2\pi \frac{a}{d}\cdot b}{\frac{n}{d}}}\right)=2$$( Since $\gcd(\frac{n}{d} , \frac{a}{d})=1$).

so , $\prod_{b=1}^{b=n} \left(1+e^{\frac{i2\pi ab}{n}}\right)=2^{d}$ , hence $$\left(\prod_{a=1}^{2015}\prod_{b=1}^{2015}\left(1+e^{2\pi iab/2015}\right)\right)=\log_{2}\left(\prod_{a=1}^{n} 2^{\gcd(a,n)}\right)=\log_{2}\left(2^{\sum_{a=1}^{n} \gcd(a,n)}\right)=\sum_{a=1}^{n} \gcd(a,n)$$
.

for $n=2015$ it is just nothing but $(5+\phi(5))(13+\phi(13))(31+\phi(30))=\boxed{13725}$. $\square$
This post has been edited 1 time. Last edited by lifeismathematics, Apr 23, 2024, 2:50 PM
Z K Y
The post below has been deleted. Click to close.
This post has been deleted. Click here to see post.
Mathandski
738 posts
#28
Y by
$          $
Attachments:
Z K Y
The post below has been deleted. Click to close.
This post has been deleted. Click here to see post.
Sagnik123Biswas
420 posts
#29
Y by
We can simplify this as $\log_2{\prod_{i=1}^{n}2^{\gcd(i, 2015)}} = \sum_{i=1}^{2015}\gcd(i, 2015) = \sum_{d}\frac{2015}{d} \phi (d) = 2015(1 + \frac{4}{5})(1 + \frac{12}{13})(1 + \frac{30}{31}) = 13725$
Z K Y
The post below has been deleted. Click to close.
This post has been deleted. Click here to see post.
smileapple
1010 posts
#30
Y by
For simplicity set $\zeta=e^{2\pi i/2015}$. Then note that $\zeta^a$ is a primitive $\frac{2015}d$th root of unity where $d=\gcd(a,2015)$, so that \[\prod_{b=1}^{2015}(1+\zeta^{ab})=\left(\prod_{b=1}^{\frac{2015}{d}}(1+\zeta^{ab})\right)^{d}=((-1)^{\frac{2015}{d}}((-1)^{\frac{2015}{d}}-1))^{d}=2^d.\]Our desired value thus reduces to \[ \log_2\left(\prod_{a=1}^{2015} \prod_{b=1}^{2015} (1+\zeta^{ab})\right)=\sum_{a=1}^{2015}\gcd(a,2015)=\sum_{d\mid 2015}\varphi(d)\left(\frac{2015}d\right).\]Note that the product on the right hand side is a Dirichlet convolution and is therefore multiplicative. Since $2015=5\cdot13\cdot31$, computation then yields an answer of $\boxed{13725}$. $\blacksquare$
Z K Y
N Quick Reply
G
H
=
a